2021 Fall AMC 10B Problems/Problem 5

Revision as of 00:55, 23 November 2021 by Kingravi (talk | contribs)

Problem 5

Let $n=8^{2022}$. Which of the following is equal to $\frac{n}{4}?$

$(\textbf{A})\: 4^{1010}\qquad(\textbf{B}) \: 2^{2022}\qquad(\textbf{C}) \: 8^{2018}\qquad(\textbf{D}) \: 4^{3031}\qquad(\textbf{E}) \: 4^{3032}$

Solution 1

We have \[n=8^{2022}=  \left(8^\frac{2}{3}\right)^{2022}=4^{3033}.\] Therefore, \[\frac{n}4=\boxed{(\textbf{E})\:4^{3032}}.\]

~kingofpineapplz


See Also

2021 Fall AMC 10B (ProblemsAnswer KeyResources)
Preceded by
Problem 4
Followed by
Problem 6
1 2 3 4 5 6 7 8 9 10 11 12 13 14 15 16 17 18 19 20 21 22 23 24 25
All AMC 10 Problems and Solutions

The problems on this page are copyrighted by the Mathematical Association of America's American Mathematics Competitions. AMC logo.png